JBMO 2018 - Θέματα - Λύσεις

Συντονιστές: cretanman, ΔΗΜΗΤΡΗΣ ΙΩΑΝΝΟΥ, socrates

Άβαταρ μέλους
Demetres
Γενικός Συντονιστής
Δημοσιεύσεις: 8989
Εγγραφή: Δευ Ιαν 19, 2009 5:16 pm
Τοποθεσία: Λεμεσός/Πύλα
Επικοινωνία:

JBMO 2018 - Θέματα - Λύσεις

#1

Μη αναγνωσμένη δημοσίευση από Demetres » Πέμ Ιουν 21, 2018 3:07 pm

Πρόβλημα 1: Να βρείτε όλα τα ζεύγη των ακεραίων (m, n) που ικανοποιούν την εξίσωση m^5-n^5=16mn.

Πρόβλημα 2: Θεωρούμε n τριψήφιους αριθμούς με τις παρακάτω ιδιότητες:

(1) Όλα τα ψηφία τους είναι διαφορετικά από το 0.
(2) Το άθροισμα των ψηφίων κάθε αριθμού είναι 9.
(3) Τα ψηφία των μονάδων δύο οποιονδήποτε αριθμών είναι διαφορετικά.
(4) Τα ψηφία των δεκάδων δύο οποιονδήποτε αριθμών είναι διαφορετικά.
(5) Τα ψηφία των εκατοντάδων δύο οποιονδήποτε αριθμών είναι διαφορετικά.

Να βρείτε τη μέγιστη δυνατή τιμή του n.

Πρόβλημα 3: Έστω k>1 ένας θετικός ακέραιος και n>2018 ένας περιττός θετικός ακέραιος. Οι μη μηδενικοί ρητοί αριθμοί x_1, x_2, \ldots, x_n δεν είναι όλοι ίσοι μεταξύ τους και ικανοποιούν την

\displaystyle {{x}_{1}}+\frac{k}{{{x}_{2}}}={{x}_{2}}+\frac{k}{{{x}_{3}}}={{x}_{3}}+\frac{k}{{{x}_{4}}}=\cdots ={{x}_{n-1}}+\frac{k}{{{x}_{n}}}={{x}_{n}}+\frac{k}{{{x}_{1}}}\, \cdot

Να βρείτε:

(α) το γινόμενο x_1 x_2\ldots x_n συναρτήσει των k και n
(β) την ελάχιστη τιμή του k, ώστε να υπάρχουν n, x_1, x_2, \ldots, x_n που να ικανοποιούν τις δοσμένες συνθήκες.

Πρόβλημα 4: Έστω ABC οξυγώνιο τρίγωνο. Τα σημεία A', B', C' είναι τα συμμετρικά των κορυφών A, B και C ως προς τις πλευρές BC, CA και AB, αντίστοιχα. Οι περιγεγραμμένοι κύκλοι των τριγώνων ABB' και ACC' τέμνονται για δεύτερη φορά στο σημείο A_1. Τα σημεία B_1 και C_1 ορίζονται με ανάλογο τρόπο. Να αποδείξετε ότι οι ευθείες AA_1, BB_1 και CC_1 διέρχονται από το ίδιο σημείο.



Λέξεις Κλειδιά:
Datis-Kalali
Δημοσιεύσεις: 117
Εγγραφή: Δευ Δεκ 12, 2016 5:33 pm
Τοποθεσία: Λευκωσία

Re: JBMO 2018 - Θέματα - Λύσεις

#2

Μη αναγνωσμένη δημοσίευση από Datis-Kalali » Πέμ Ιουν 21, 2018 4:08 pm

1) Θέτουμε m=dm_0 , n=dn_0 όπου (m,n)=d
d^3({m_0}^5-{n_0}^5 )=16m_0n_0 (1)
Αφού (m_0,n_0)=1
{m_0}^5-{n_0}^5 \vert 16
οπότε πρεέι να λύσουμε τις εξισώσεις
{m_0}^5-{n_0}^5=1, που είναι άτοπο απο το θεώρημα Mihaelescu
ή
{m_0}^5-{n_0}^5=2 που έχει μοναδική λύση (m_0,n_0)=(1,-1)
ή
{m_0}^5-{n_0}^5=-2 που έχει μοναδική λύση (m_0,n_0)=(-1,1)
ή
{m_0}^5-{n_0}^5=2^k
k=2,3,4
που δεν έχουν λύσεις, καθώς x^5-y^5 αυξάνεται γρήγορα (function growth)
η μοναδική περίπτωση είανι m_0=0 και n_0=0
Δηλαδή αντικαθηστώντας στο (1), έχουμε d=2
και έτσι:
(m,n)=(2,-2),(-2,2),(0,0)


Άβαταρ μέλους
Διονύσιος Αδαμόπουλος
Δημοσιεύσεις: 807
Εγγραφή: Σάβ Μαρ 19, 2016 5:11 pm
Τοποθεσία: Πύργος Ηλείας

Re: JBMO 2018 - Θέματα - Λύσεις

#3

Μη αναγνωσμένη δημοσίευση από Διονύσιος Αδαμόπουλος » Παρ Ιουν 22, 2018 11:12 am

Demetres έγραψε:
Πέμ Ιουν 21, 2018 3:07 pm

Πρόβλημα 4: Έστω ABC οξυγώνιο τρίγωνο. Τα σημεία A', B', C' είναι τα συμμετρικά των κορυφών A, B και C ως προς τις πλευρές BC, CA και AB, αντίστοιχα. Οι περιγεγραμμένοι κύκλοι των τριγώνων ABB' και ACC' τέμνονται για δεύτερη φορά στο σημείο A_1. Τα σημεία B_1 και C_1 ορίζονται με ανάλογο τρόπο. Να αποδείξετε ότι οι ευθείες AA_1, BB_1 και CC_1 διέρχονται από το ίδιο σημείο.
Θεωρούμε αρχικά συμμετρική αντιστροφή με πόλο το A και δύναμη AB\cdot AC (συμμετρική είναι ως προς την διχοτόμο της γωνίας A).

Από τον ορισμό αυτής της αντιστροφής το B πάει στο C και το ανάποδο.

Λόγω του ότι οι AB' και AC' είναι ισογώνιες και AC'=AC και AB'=AB, με αποτέλεσμα AC'\cdot AB'=AB\cdot AC, που είναι η δύναμη, έχουμε πως τα B' και C' αντιστρέφονται μεταξύ τους.

Άρα οι περιγεγραμμένοι κύκλοι των ABB' και ACC' γίνονται οι CC' και BB' αντίστοιχα. Το σημείο τομής αυτών δηλαδή είναι το αντίστροφο του A_1.

Όμως οι CC' και οι BB' ταυτίζονται με τα ύψη των αντίστοιχων κορυφών από τις C, B, επομένως το αντίστροφο του A_1 είναι το ορθόκεντρο του ABC. Όμως ξέρουμε από τις ιδιότητες αυτής της αντιστροφής πως οι AH και AA_1 είναι ισογώνιες. Άρα η AA_1 ταυτίζεται με την ευθεία AO, όπου O το περίκεντρο του ABC (αφού τα O, H είναι ισογώνια).

Όμοια η BB_1 και η CC_1 ταυτίζονται με τις BO και CO, επομένως οι τρεις ευθείες μας διέρχονται από το ίδιο σημείο, το O.
JBMO 2018 P4.png
JBMO 2018 P4.png (41.61 KiB) Προβλήθηκε 3315 φορές
Προστέθηκε το σχήμα (δεν είναι ολόκληρο, αλλά φαίνονται τα βασικά σημεία της λύσης)!
τελευταία επεξεργασία από Διονύσιος Αδαμόπουλος σε Παρ Ιουν 22, 2018 8:14 pm, έχει επεξεργασθεί 1 φορά συνολικά.


Houston, we have a problem!
Άβαταρ μέλους
Demetres
Γενικός Συντονιστής
Δημοσιεύσεις: 8989
Εγγραφή: Δευ Ιαν 19, 2009 5:16 pm
Τοποθεσία: Λεμεσός/Πύλα
Επικοινωνία:

Re: JBMO 2018 - Θέματα - Λύσεις

#4

Μη αναγνωσμένη δημοσίευση από Demetres » Παρ Ιουν 22, 2018 11:45 am

Demetres έγραψε:
Πέμ Ιουν 21, 2018 3:07 pm

Πρόβλημα 4: Έστω ABC οξυγώνιο τρίγωνο. Τα σημεία A', B', C' είναι τα συμμετρικά των κορυφών A, B και C ως προς τις πλευρές BC, CA και AB, αντίστοιχα. Οι περιγεγραμμένοι κύκλοι των τριγώνων ABB' και ACC' τέμνονται για δεύτερη φορά στο σημείο A_1. Τα σημεία B_1 και C_1 ορίζονται με ανάλογο τρόπο. Να αποδείξετε ότι οι ευθείες AA_1, BB_1 και CC_1 διέρχονται από το ίδιο σημείο.

Βάζω και την δική μου λύση για το 4. (Ας βάλει κάποιος ένα σχήμα. :) )

Έχουμε \angle AA_1B = \angle AB'B = \angle ABB' = 90^{\circ} - \hat{A} αφού η BB' είναι κάθετη στην AC. Ομοίως \angle AA_1C = 90^{\circ} - \hat{A}. Άρα BA_1C = 180^{\circ} - 2\hat{A} και επειδή BOC = 2\hat{A}, όπου O το περίκεντρο του ABC, τότε τα O,B,A_1,C είναι ομοκυκλικά.

Τότε \angle OA_1B = \angle OCB = 90^{\circ} - \hat{A}. (Από το ισόπλευρο τρίγωνο OCB.)

Αφού \angle OA_1B = \angle AA_1B τα O,A,A_1 είναι συνευθειακά, δηλαδή το O ανήκει πάνω στην AA_1. Ομοίως ανήκει και στις BB_1 και CC_1


Άβαταρ μέλους
Διονύσιος Αδαμόπουλος
Δημοσιεύσεις: 807
Εγγραφή: Σάβ Μαρ 19, 2016 5:11 pm
Τοποθεσία: Πύργος Ηλείας

Re: JBMO 2018 - Θέματα - Λύσεις

#5

Μη αναγνωσμένη δημοσίευση από Διονύσιος Αδαμόπουλος » Παρ Ιουν 22, 2018 1:00 pm

Demetres έγραψε:
Πέμ Ιουν 21, 2018 3:07 pm

Πρόβλημα 2: Θεωρούμε n τριψήφιους αριθμούς με τις παρακάτω ιδιότητες:

(1) Όλα τα ψηφία τους είναι διαφορετικά από το 0.
(2) Το άθροισμα των ψηφίων κάθε αριθμού είναι 9.
(3) Τα ψηφία των μονάδων δύο οποιονδήποτε αριθμών είναι διαφορετικά.
(4) Τα ψηφία των δεκάδων δύο οποιονδήποτε αριθμών είναι διαφορετικά.
(5) Τα ψηφία των εκατοντάδων δύο οποιονδήποτε αριθμών είναι διαφορετικά.

Να βρείτε τη μέγιστη δυνατή τιμή του n.
Προφανώς τα ψηφία 9, 8 δεν μπορούν να χρησιμοποιηθούν.

Αν χρησιμοποιηθεί το ψηφίο 7, τότε μπορεί να χρησιμοποιηθεί μόνο ως (7, 1, 1) (ή με κάποια αναδιάταξη αυτού). Αυτός ο συνδυασμός όμως μπορεί να χρησιμοποιηθεί μόνο μια φορά, καθώς αν εμφανιζόταν και δεύτερη, τότε σε μια θέση θα εμφανιζόταν ξανά το ψηφίο 1. Άρα το 7 μπορεί να χρησιμοποιηθεί μόνο μια φορά.

Παρατηρούμε επίσης πως κάθε άλλο ψηφίο μπορεί να εμφανιστεί το πολύ 3 φορές (το πολύ μια φορά σε εκατοντάδες, δεκάδες και μονάδες).

Ακόμη το πλήθος όλων των ψηφίων που χρησιμοποιούνται πρέπει να είναι πολλαπλάσιο του 3 (αφού έχουμε τριψήφιους αριθμούς).

Από τα παραπάνω μπορούν να χρησιμοποιηθούν το πολύ 1+6\cdot 3=19 ψηφία (3 φορές τα ψηφία από 1 έως 6 και μια φορά το 7). Προφανώς αυτό δεν γίνεται άρα ένα πρώτο ανώτερο όριο είναι τα 18 ψηφία, δηλαδή να έχουμε n=6.

Για να ισχύει αυτό πρέπει το ψηφίο 6 να χρησιμοποιηθεί τουλάχιστον 2 φορές. Πρέπει να τονιστεί πως το ψηφίο 6 μπορεί να εμφανιστεί μόνο ως (6, 1, 2) (ή με κάποια αναδιάταξη αυτού).

Έστω πως το 6 χρησιμοποιείται στους τριψήφιους 612 και 261 (δεν παίζει ρόλο η σειρά απλά το ότι εμφανίζεται το 1 το 2 και το 6 δύο φορές).

Το ψηφίο 7 επομένως δεν μπορεί να χρησιμοποιηθεί, καθώς έχουν χρησιμοποιηθεί ήδη 2 άσσοι.

Άρα για να έχουμε 18 ψηφία πρέπει όλα τα ψηφία από το 1 έως το 6 να χρησιμοποιηθούν 3 φορές.

Αφού το 6 θα χρησιμοποιηθεί 3 φορές, θα χρησιμοποιηθεί επιπλέον 3 φορές το 1 και το 2.

Το 5 μπορεί να εμφανιστεί μόνο ως (5, 2, 2) και (5, 3, 1). Αυτό όμως δεν γίνεται καθώς έχει ήδη χρησιμοποιηθεί ο μέγιστος αριθμός ψηφίων 1 και 2.

Άρα δεν γίνεται να έχουμε 18 ψηφία, δηλαδή έχουμε το πολύ 15 ψηφία, δηλαδή το πολύ 5 τριψήφιους.

Μια πεντάδα είναι η εξής:

135

513

252

441

324

Άρα πράγματι το μέγιστο n είναι n=5.


Houston, we have a problem!
Άβαταρ μέλους
Demetres
Γενικός Συντονιστής
Δημοσιεύσεις: 8989
Εγγραφή: Δευ Ιαν 19, 2009 5:16 pm
Τοποθεσία: Λεμεσός/Πύλα
Επικοινωνία:

Re: JBMO 2018 - Θέματα - Λύσεις

#6

Μη αναγνωσμένη δημοσίευση από Demetres » Παρ Ιουν 22, 2018 1:07 pm

Ένας άλλος τρόπος για να δούμε ότι έχουμε το πολύ 5 αριθμούς είναι ο εξής:

Αν έχουμε k αριθμούς, τότε το άθροισμα των ψηφίων τους ισούται με 9k. Όμως το άθροισμα των ψηφίων ισούται τουλάχιστον με \displaystyle 3(1+2+\cdots + k) = \frac{3k(k+1)}{2}. Άρα \displaystyle  9k \geqslant \frac{3k(k+1)}{2} που δίνει k \leqslant 5.


Datis-Kalali
Δημοσιεύσεις: 117
Εγγραφή: Δευ Δεκ 12, 2016 5:33 pm
Τοποθεσία: Λευκωσία

Re: JBMO 2018 - Θέματα - Λύσεις

#7

Μη αναγνωσμένη δημοσίευση από Datis-Kalali » Παρ Ιουν 22, 2018 1:32 pm

Demetres έγραψε:
Πέμ Ιουν 21, 2018 3:07 pm
Πρόβλημα 3: Έστω k>1 ένας θετικός ακέραιος και n>2018 ένας περιττός θετικός ακέραιος. Οι μη μηδενικοί ρητοί αριθμοί x_1, x_2, \ldots, x_n δεν είναι όλοι ίσοι μεταξύ τους και ικανοποιούν την

\displaystyle {{x}_{1}}+\frac{k}{{{x}_{2}}}={{x}_{2}}+\frac{k}{{{x}_{3}}}={{x}_{3}}+\frac{k}{{{x}_{4}}}=\cdots ={{x}_{n-1}}+\frac{k}{{{x}_{n}}}={{x}_{n}}+\frac{k}{{{x}_{1}}}\, \cdot

Να βρείτε:

(α) το γινόμενο x_1 x_2\ldots x_n συναρτήσει των k και n
(β) την ελάχιστη τιμή του k, ώστε να υπάρχουν n, x_1, x_2, \ldots, x_n που να ικανοποιούν τις δοσμένες συνθήκες.
Γράφουμε x_{i+1}-x_{i}=k(\frac{1}{x_{i+1}}-\frac{1}{x_{i+2}}) - Σχέση (1)
Ισχυρισμός: Δεν μπορούμε να έχουμε x_i=x_{i+1}
Απόδειξη:
Εαν έχουμε x_i=x_{i+1} τότε απο την σχέση (1) έχουμε x_{i+1}=x_{i+2}, όποτε επαπγωγικά όλοι οι αριθμοί είναι ίσες, άτοπο!
Τωρά η σχέση (1) γράφεται
x_{i+1}-x_{i}=k(\frac{x_{i+2}-x_{i+1}}{x_{i+1}x_{i+2}})
\Rightarrow x_{i+1}x_{i+2}=k(\frac{x_{i+2}-x_{i+1}}{x_{i+1}-x_{i}})
οπότε πολλαπλασιάζοντας κατά μέλη οι ισότητες για i=1,2,...,n
(όπου x_i \neq x_{i+1})
(x_{1}x_{2}...x_{n})^{2}=k^n
\Rightarrow x_1 x_2\ldots x_n=k^{\frac{n}{2}}
β) Αφού x_1 x_2\ldots x_n είανι ρητό, πρέπει
k^{\frac{n}{2}} να είναι ρητό.
Αλλά αφού n είανι περιττό, πρέπει το κ να είανι τέλειο τετράγωνο ακεραίου.
Δηλαδή k \ge 4
Για k=4, εύκολα μπορύμε να βρούμε παράδειγμα


Άβαταρ μέλους
gbaloglou
Επιμελητής
Δημοσιεύσεις: 3341
Εγγραφή: Παρ Φεβ 27, 2009 10:24 pm
Τοποθεσία: Θεσσαλονικη
Επικοινωνία:

Re: JBMO 2018 - Θέματα - Λύσεις

#8

Μη αναγνωσμένη δημοσίευση από gbaloglou » Παρ Ιουν 22, 2018 3:32 pm

Στο Πρόβλημα 2 ακολούθησα 'πορεία εξόντωσης', παρατηρώντας αρχικά ότι αποκλείονται τα ψηφία 9 και 8, και εξετάζοντας μετά τις περιπτώσεις όπου το μέγιστο ψηφίο είναι 7, 6, κλπ Μου ξέφευγαν όμως περιπτώσεις, και κατέληξα να αποδείξω ότι n=4! Διόρθωσα το λάθος μου πριν δω τις λύσεις του Διονύση και του Δημήτρη, καθώς προσπαθούσα να κατασκευάσω έναν πίνακα λύσεων και 'μεγίστων', μη ισοδυνάμων περιπτώσεων. Επισυνάπτω αυτόν τον πίνακα ... χωρίς να είμαι και 100% σίγουρος ότι είναι πλήρης, πιστεύοντας όμως ότι το παράδειγμα 5 αριθμών που παρέθεσε ο Διονύσης είναι ουσιαστικά μοναδικό.

ΕΠΕΞΕΡΓΑΣΙΑ 24-6-2018 1:15 μμ: αλλαγή πίνακα λύσεων

JBMO-2018.png
JBMO-2018.png (10.96 KiB) Προβλήθηκε 3067 φορές
τελευταία επεξεργασία από gbaloglou σε Κυρ Ιουν 24, 2018 1:15 pm, έχει επεξεργασθεί 1 φορά συνολικά.


Γιώργος Μπαλόγλου -- κρυσταλλογράφω άρα υπάρχω

Ὁρᾷς, τὸ κάλλος ὅσσον ἐστὶ τῆς λίθου, ἐν ταῖς ἀτάκτοις τῶν φλεβῶν εὐταξίαις. -- Παλατινή Ανθολογία 9.695 -- Ιδού του πετραδιού η άμετρη ομορφιά, μεσ' των φλεβών τις άναρχες πειθαρχίες.
Άβαταρ μέλους
Διονύσιος Αδαμόπουλος
Δημοσιεύσεις: 807
Εγγραφή: Σάβ Μαρ 19, 2016 5:11 pm
Τοποθεσία: Πύργος Ηλείας

Re: JBMO 2018 - Θέματα - Λύσεις

#9

Μη αναγνωσμένη δημοσίευση από Διονύσιος Αδαμόπουλος » Παρ Ιουν 22, 2018 8:24 pm

Demetres έγραψε:
Παρ Ιουν 22, 2018 11:45 am
Demetres έγραψε:
Πέμ Ιουν 21, 2018 3:07 pm

Πρόβλημα 4: Έστω ABC οξυγώνιο τρίγωνο. Τα σημεία A', B', C' είναι τα συμμετρικά των κορυφών A, B και C ως προς τις πλευρές BC, CA και AB, αντίστοιχα. Οι περιγεγραμμένοι κύκλοι των τριγώνων ABB' και ACC' τέμνονται για δεύτερη φορά στο σημείο A_1. Τα σημεία B_1 και C_1 ορίζονται με ανάλογο τρόπο. Να αποδείξετε ότι οι ευθείες AA_1, BB_1 και CC_1 διέρχονται από το ίδιο σημείο.

Βάζω και την δική μου λύση για το 4. (Ας βάλει κάποιος ένα σχήμα. :) )

Έχουμε \angle AA_1B = \angle AB'B = \angle ABB' = 90^{\circ} - \hat{A} αφού η BB' είναι κάθετη στην AC. Ομοίως \angle AA_1C = 90^{\circ} - \hat{A}. Άρα BA_1C = 180^{\circ} - 2\hat{A} και επειδή BOC = 2\hat{A}, όπου O το περίκεντρο του ABC, τότε τα O,B,A_1,C είναι ομοκυκλικά.

Τότε \angle OA_1B = \angle OCB = 90^{\circ} - \hat{A}. (Από το ισόπλευρο τρίγωνο OCB.)

Αφού \angle OA_1B = \angle AA_1B τα O,A,A_1 είναι συνευθειακά, δηλαδή το O ανήκει πάνω στην AA_1. Ομοίως ανήκει και στις BB_1 και CC_1
Χμμμ τελικά έβγαινε πιο απλά από ότι φανταζόμουν... Δεν μπορούσα να κρατηθώ όμως και να μην χρησιμοποιήσω ΑΝΤΙΣΤΡΟΦΗ σε αυτό το όμορφο πρόβλημα.

Ένα σχήμα στην ωραία αυτή λύση του κύριου Δημήτρη:
JBMO 2018 P4 v2.png
JBMO 2018 P4 v2.png (48.05 KiB) Προβλήθηκε 3311 φορές


Houston, we have a problem!
Άβαταρ μέλους
Demetres
Γενικός Συντονιστής
Δημοσιεύσεις: 8989
Εγγραφή: Δευ Ιαν 19, 2009 5:16 pm
Τοποθεσία: Λεμεσός/Πύλα
Επικοινωνία:

Re: JBMO 2018 - Θέματα - Λύσεις

#10

Μη αναγνωσμένη δημοσίευση από Demetres » Κυρ Ιουν 24, 2018 11:03 am

Datis-Kalali έγραψε:
Παρ Ιουν 22, 2018 1:32 pm
Demetres έγραψε:
Πέμ Ιουν 21, 2018 3:07 pm
Πρόβλημα 3: Έστω k>1 ένας θετικός ακέραιος και n>2018 ένας περιττός θετικός ακέραιος. Οι μη μηδενικοί ρητοί αριθμοί x_1, x_2, \ldots, x_n δεν είναι όλοι ίσοι μεταξύ τους και ικανοποιούν την

\displaystyle {{x}_{1}}+\frac{k}{{{x}_{2}}}={{x}_{2}}+\frac{k}{{{x}_{3}}}={{x}_{3}}+\frac{k}{{{x}_{4}}}=\cdots ={{x}_{n-1}}+\frac{k}{{{x}_{n}}}={{x}_{n}}+\frac{k}{{{x}_{1}}}\, \cdot

Να βρείτε:

(α) το γινόμενο x_1 x_2\ldots x_n συναρτήσει των k και n
(β) την ελάχιστη τιμή του k, ώστε να υπάρχουν n, x_1, x_2, \ldots, x_n που να ικανοποιούν τις δοσμένες συνθήκες.
Γράφουμε x_{i+1}-x_{i}=k(\frac{1}{x_{i+1}}-\frac{1}{x_{i+2}}) - Σχέση (1)
Ισχυρισμός: Δεν μπορούμε να έχουμε x_i=x_{i+1}
Απόδειξη:
Εαν έχουμε x_i=x_{i+1} τότε απο την σχέση (1) έχουμε x_{i+1}=x_{i+2}, όποτε επαπγωγικά όλοι οι αριθμοί είναι ίσες, άτοπο!
Τωρά η σχέση (1) γράφεται
x_{i+1}-x_{i}=k(\frac{x_{i+2}-x_{i+1}}{x_{i+1}x_{i+2}})
\Rightarrow x_{i+1}x_{i+2}=k(\frac{x_{i+2}-x_{i+1}}{x_{i+1}-x_{i}})
οπότε πολλαπλασιάζοντας κατά μέλη οι ισότητες για i=1,2,...,n
(όπου x_i \neq x_{i+1})
(x_{1}x_{2}...x_{n})^{2}=k^n
\Rightarrow x_1 x_2\ldots x_n=k^{\frac{n}{2}}
β) Αφού x_1 x_2\ldots x_n είανι ρητό, πρέπει
k^{\frac{n}{2}} να είναι ρητό.
Αλλά αφού n είανι περιττό, πρέπει το κ να είανι τέλειο τετράγωνο ακεραίου.
Δηλαδή k \ge 4
Για k=4, εύκολα μπορύμε να βρούμε παράδειγμα

Στο (α) υπάρχει και η περίπτωση x_1 x_2 \cdots x_n = -\sqrt{k^n}.

Για το (β) ένα παράδειγμα με k=4 και n=3 είναι το x_1 = 2,x_2=-1,x_3=-4.

Ας δούμε όμως και πως μπορούμε να βρούμε το παράδειγμα: Δοκιμάζουμε n=3 όπου ελπίζουμε τα πράγματα να είναι πιο απλά. Γράφουμε x_1 = 2a,x_2=2b,x_3=2c και η συνθήκη γίνεται

\displaystyle  a + \frac{1}{b} = b + \frac{1}{c} = c+\frac{1}{a}

όπου abc = 1. Ας γράψουμε m = a + \frac{1}{b}. Οπότε είναι b = \frac{1}{m-a}. Επίσης m = b + \frac{1}{c} = b+ba που δίνει b = \frac{m}{1+a}. Άρα \frac{1}{m-a} = \frac{m}{1+a} και με πράξεις καταλήγουμε στο (m+1)(m-1-a) = 0. Ομοίως παίρνουμε και (m+1)(m-1-b) = (m+1)(m-1-c) = 0. Πρέπει m=-1 αφού αλλιώς θα είχαμε a=b=c=m-1.

Με m=-1 (σχεδόν) οτιδήποτε δοκιμάσουμε δουλεύει. Π.χ. εγώ ξεκίνησα με a=1 που δίνει b=-1/2 και c = -2.


Για δείτε και αυτό εδώ. :no:


Άβαταρ μέλους
Demetres
Γενικός Συντονιστής
Δημοσιεύσεις: 8989
Εγγραφή: Δευ Ιαν 19, 2009 5:16 pm
Τοποθεσία: Λεμεσός/Πύλα
Επικοινωνία:

Re: JBMO 2018 - Θέματα - Λύσεις

#11

Μη αναγνωσμένη δημοσίευση από Demetres » Κυρ Ιουν 24, 2018 3:54 pm

Demetres έγραψε:
Κυρ Ιουν 24, 2018 11:03 am
Για το (β) ένα παράδειγμα με k=4 και n=3 είναι το x_1 = 2,x_2=-1,x_3=-4.
Όπως με ενημέρωσε ο Διονύσης, η εκφώνηση ζητάει παράδειγμα με n > 2018. :oops:

Ευτυχώς το πιο πάνω διορθώνεται εύκολα για να δώσει για n=2019 το παράδειγμα x_{3k+1} = 2,x_{3k+2} = -1, x_{3k+3} = -4 για 0 \leqslant k \leqslant 672.


Άβαταρ μέλους
Demetres
Γενικός Συντονιστής
Δημοσιεύσεις: 8989
Εγγραφή: Δευ Ιαν 19, 2009 5:16 pm
Τοποθεσία: Λεμεσός/Πύλα
Επικοινωνία:

Re: JBMO 2018 - Θέματα - Λύσεις

#12

Μη αναγνωσμένη δημοσίευση από Demetres » Δευ Ιουν 25, 2018 11:12 am

Datis-Kalali έγραψε:
Πέμ Ιουν 21, 2018 4:08 pm
1) Θέτουμε m=dm_0 , n=dn_0 όπου (m,n)=d
d^3({m_0}^5-{n_0}^5 )=16m_0n_0 (1)
Αφού (m_0,n_0)=1
{m_0}^5-{n_0}^5 \vert 16
Συμπληρώνω διαφορετικά την λύση μιας και δεν χρειάζεται η αναφορά στο θεώρημα Mihailescu.

Θα βρούμε όλα τα ζεύγη μη μηδενικών ακεραίων (x,y) ώστε |x^5 - y^5| \in \{1,2,4,8,16\}. Αλλάζοντας πρόσημα μπορώ να υποθέσω ότι x > 0. Αν y < 0, τότε x^5 - y^5 > 32 εκτός και αν x=1,y=-1. Για x=1,y=-1 έχουμε x^5 - y^5 = 2.

Μπορούμε λοιπόν τώρα να υποθέσουμε x > y > 0. Τότε x^5 - y^5 \geqslant (y+1)^5 - y^5 = 5y^4 + 10y^3+10y^2 + 5y+1 \geqslant 31.

Άρα μοναδικές λύσεις είναι οι (1,-1) και (-1,1).

Η συνέχεια όπως στην λύση του Datis.


AΝΔΡΕΑΣ ΒΑΡΒΕΡΑΚΗΣ
Επιμελητής
Δημοσιεύσεις: 1172
Εγγραφή: Τετ Δεκ 31, 2008 8:07 pm
Τοποθεσία: ΗΡΑΚΛΕΙΟ ΚΡΗΤΗΣ

Re: JBMO 2018 - Θέματα - Λύσεις

#13

Μη αναγνωσμένη δημοσίευση από AΝΔΡΕΑΣ ΒΑΡΒΕΡΑΚΗΣ » Δευ Ιουν 25, 2018 11:31 am

Άλλη μία ιδέα για το 4.
Από την ομοιότητα των ισοσκελών τριγώνων ABB', ACC' με λόγο ομοιότητας \dfrac{AB}{AC} και καθώς τα αντίστοιχα περίκεντρα των ABB', ACC' (έστω L, K) βρίσκονται επί των AC, AB ανίστοιχα (καθώς οι AC, AB είναι μεσοκάθετοι των βάσεων τους), προκύπτει η αναλογία \dfrac{AL}{AK}=\dfrac{AB}{AC}, που σημαίνει ότι η διάκεντρος LK είναι αντιπαράλληλος της BC, επομένως η κοινή χορδή AA_1 που είναι κάθετη στη διάκεντρο, είναι ισογώνια του ύψους AH, άρα διέρχεται από το περίκεντρο O του τριγώνου ABC.


Άβαταρ μέλους
Demetres
Γενικός Συντονιστής
Δημοσιεύσεις: 8989
Εγγραφή: Δευ Ιαν 19, 2009 5:16 pm
Τοποθεσία: Λεμεσός/Πύλα
Επικοινωνία:

Re: JBMO 2018 - Θέματα - Λύσεις

#14

Μη αναγνωσμένη δημοσίευση από Demetres » Δευ Ιουν 25, 2018 3:10 pm

Demetres έγραψε:
Πέμ Ιουν 21, 2018 3:07 pm
Πρόβλημα 1: Να βρείτε όλα τα ζεύγη των ακεραίων (m, n) που ικανοποιούν την εξίσωση m^5-n^5=16mn.

Δίνω μια άλλη λύση. Δεν είναι τόσο απλή αλλά η μέθοδος χρησιμοποιείται σε παρόμοια προβλήματα.

Αν m=0 ή n=0 παίρνουμε την λύση (0,0). Υποθέτουμε λοιπόν ότι m,n \neq 0.

Είναι απλό ότι m|n^5 και n|m^5. Άρα οι m,n έχουν ακριβώς τους ίδιους πρώτους διαιρέτες. Έστω p ένας πρώτος διαιρέτης των m,n. Γράφουμε a,b,c,d για τις μέγιστες τιμές ώστε τα p^a,\ldots,p^d να διαιρούν τα m,n,m^5-n^5,16mn. Προφανώς πρέπει c=d.

Αν p περιττός και a=b τότε c \geqslant 5a και d = 2a, άτοπο.

Αν p περιττός και a \neq b τότε c =  5\min\{a,b\} και d = a+b. Άρα a = 4b ή b = 4a.

Αν p=2 και a = b τότε c \geqslant 5a+1 και d = 2a+4. Άρα 2a+4 \geqslant 5a+1 που δίνει a=b=1.

Αν p=2 και a \neq b τότε c =  5\min\{a,b\} και d = a+b+4. Άρα a = 4b-4 ή b = 4a-4.

Τα πιο πάνω δίνουν ότι υπάρχουν φυσικοί r,s ώστε

(α) m = \pm rs^4 και n = \pm r^4s ή
(β) m = \pm 2rs^4 και n = \pm 2r^4s ή

Σε κάθε περίπτωση έχουμε τέσσερις επιλογές προσήμων. Η περίπτωση (β) περιλαμβάνει και την περίπτωση p=2 με a \neq b. Π.χ. η περίπτωση a=4b-4 λαμβάνεται όταν ο r είναι περιττός και η μεγαλύτερη δύναμη του 2 που διαιρεί το s ισούται με b-1.

Στην περίπτωση (α) καταλήγουμε στο \displaystyle  \pm r^5 s^{20} \pm r^{20}s^5 = 16 r^5s^5, δηλαδή  \pm s^{15} \pm r^{15} = 16. Στη περίπτωση (β) καταλήγουμε στο \pm s^{15} \pm r^{15} = 2.

Όπως και στην άλλη λύση καταλήγουμε ότι λύσεις υπάρχουν μόνο στην περίπτωση (β) και πρέπει r = \pm 1 και s = \pm 1. Δηλαδή m = \pm 2 και n = \pm 2. Ελέγχοντας τώρα τις περιπτώσεις προσήμου που δουλεύουν έχουμε τις λύσεις (2,-2) και (-2,2).


Άβαταρ μέλους
cretanman
Διαχειριστής
Δημοσιεύσεις: 4097
Εγγραφή: Πέμ Δεκ 18, 2008 12:35 pm
Τοποθεσία: Ηράκλειο Κρήτης
Επικοινωνία:

Re: JBMO 2018 - Θέματα - Λύσεις

#15

Μη αναγνωσμένη δημοσίευση από cretanman » Δευ Ιουν 25, 2018 11:57 pm

Datis-Kalali έγραψε:
Πέμ Ιουν 21, 2018 4:08 pm
1) Θέτουμε m=dm_0 , n=dn_0 όπου (m,n)=d
d^3({m_0}^5-{n_0}^5 )=16m_0n_0 (1)
Αφού (m_0,n_0)=1
{m_0}^5-{n_0}^5 \vert 16
Ένα διαφορετικό έξυπνο τελείωμα που χρησιμοποίησαν 2 (τουλάχιστον) διαγωνιζόμενοι για να απορρίψουν τις περιπτώσεις όπου |m_0^5-n_0^5|\in\{\pm 4,\pm 8,\pm 16\}.

Λήμμα: Για οποιοδήποτε ακέραιο x ισχύει x^5\equiv 0,\pm 1 \pmod{11}.
Πράγματι, αν 11|x τότε ισχύει x^5\equiv 0\pmod{11}.

Αν 11\not\vert x τότε από το μικρό θεώρημα του Fermat έχουμε x^{10}\equiv 1\pmod{11} κι έτσι x^{5}\equiv \pm 1 \pmod{11}.

Τελικά, χρησιμοποιώντας το λήμμα έχουμε m_0^5-n_0^5\equiv 0,\pm 1,\pm 2\pmod{11} κι έτσι δε μπορεί να είναι |m_0^5-n_0^5|\in\{\pm 4,\pm 8,\pm 16\}.

Μένουν οι περιπτώσεις |m_0^5-n_0^5|=1 ή |m_0^5-n_0^5|=2 και η συνέχεια όπως στη λύση του Δημήτρη.

Αλέξανδρος


Αλέξανδρος Συγκελάκης
achilleas
Γενικός Συντονιστής
Δημοσιεύσεις: 3014
Εγγραφή: Τρί Σεπ 15, 2009 3:32 pm

Re: JBMO 2018 - Θέματα - Λύσεις

#16

Μη αναγνωσμένη δημοσίευση από achilleas » Τρί Ιουν 26, 2018 12:10 am

Demetres έγραψε:
Πέμ Ιουν 21, 2018 3:07 pm
Πρόβλημα 1: Να βρείτε όλα τα ζεύγη των ακεραίων (m, n) που ικανοποιούν την εξίσωση m^5-n^5=16mn.

...
Μετά την ωραία λύση του Δημήτρη, ας παραθέσω μια άλλη (αλγεβρική) λύση. Ήλπιζα η ιδέα της διακρίνουσας να διευθετούσε συνολικά την άσκηση, αλλά αναγκάστηκα να καταφύγω σε μια παλιότερη ιδέα για να ολοκληρώσω την λύση και να χαλάσω έτσι την κομψότητα της.


Αν mn=0, τότε m=n=0. Αν mn\ne 0, τότε η εξίσωση

\displaystyle{m^3x^2-16x-n^3=0\qquad (1)}

έχει ως μια λύση την x=\dfrac{m}{n} κι άρα η διακρίνουσά της πρέπει να είναι μη αρνητική

\displaystyle{\Delta=256+4m^3n^3\geq 0,}

οπότε mn\geq -4.

Παρατηρώντας ότι εάν ένας πρώτος διαιρεί τον m, τότε θα διαιρεί και τον n, και αντίστροφα, το γινόμενο mn δεν μπορεί να ισούται με -2 ή με -3.

Έτσι, αν mn<0, περιορίζουμε τα πιθανά ζευγάρια στα (-1,1), (1,-1) με γινόμενο -1, και (-2,2), (2,-2) με γινόμενο -4. Εύκολα βλέπουμε ότι μόνο το (-2,2) μας δίνει λύση.

Ας θεωρήσουμε ότι mn>0, οπότε και \dfrac{m}{n}>0. Επαναλαμβάνοντας την ιδέα από αυτή τη λύση, έχουμε

\begin{aligned}\notag 
|m-n|&=\dfrac{16mn}{m^4+m^3n+m^2n^2+mn^3+n^4}\\\notag 
          &\leq \dfrac{16m^2n^2}{m^4+m^3n+m^2n^2+mn^3+n^4}\\\notag 
          &=\dfrac{16}{\frac{m^2}{n^2}+\frac{m}{n}+1+\frac{n}{m}+\frac{n^2}{m^2}}\\\notag 
          &\leq \dfrac{16}{5}\notag 
\end{aligned}

Αφού οι m,n είναι και οι δύο άρτιοι ή και οι δύο περιττοί, η διαφορά τους είναι άρτιος. Αναγκαστικά, λοιπόν, θα είναι |m-n|=2.

Τότε θα έχουμε |mn|\geq 3 (με |m|\geq 1, |n|\geq 3 ή |m|\geq 3, |n|\geq 1), και

\begin{aligned}\notag 
16mn&=|m^5-n^5|\\\notag 
&=2(m^4+m^3n+m^2n^2+mn^3+n^4)\\\notag 
&>2mn(m^2+mn+n^2)\\\notag 
&\geq 2mn(1+3+9)\\\notag 
&=26mn,\notag 
\end{aligned}

άτοπο.

Συνοψίζοντας, οι λύσεις της εξίσωσης είναι (m,n)=(0,0) ή (-2,2).

Φιλικά,

Αχιλλέας


Άβαταρ μέλους
gbaloglou
Επιμελητής
Δημοσιεύσεις: 3341
Εγγραφή: Παρ Φεβ 27, 2009 10:24 pm
Τοποθεσία: Θεσσαλονικη
Επικοινωνία:

Re: JBMO 2018 - Θέματα - Λύσεις

#17

Μη αναγνωσμένη δημοσίευση από gbaloglou » Τρί Ιουν 26, 2018 1:45 pm

Μία ακόμη λύση για το Πρόβλημα 1, με function growth (Datis-Kalali) χωρίς καμμία αριθμοθεωρητική θεώρηση:

Εξετάζοντας όλες τις δυνατές περιπτώσεις θετικών-αρνητικών βλέπουμε ότι το πρόβλημα ανάγεται στις εξισώσεις m^5+n^5=16mn & m^5-n^5=16mn για m\geq n\geq 0. Η περίπτωση m=n δίνει m=n=2 ή m=n=0 για την πρώτη εξίσωση και m=n=0 για την δεύτερη εξίσωση, οδηγώντας στις λύσεις (2,-2) και (0,0) της δοθείσης. Υποθέτουμε λοιπόν στο εξής m>n>0 και δείχνουμε ότι οι εξισώσεις m^5+n^5=16mn & m^5-n^5=16mn δεν έχουν τέτοιες (ακέραιες) λύσεις.

Προκύπτει εύκολα -- είτε με χρήση παραγώγων (f'(x)>0) είτε χωρίς (x_1<x_2\rightarrow f(x_1)<f(x_2)) -- ότι η συνάρτηση f(m)=\dfrac{m^5\pm n^5}{16mn} είναι αύξουσα ως προς m, οπότε, από f(m)\geq f(n+1), αρκεί να δειχθεί η f(n+1)>1, ισοδύναμη είτε προς την 2n^5+5n^4+10n^3-6n^2-11n+1>0 (αληθής για n>0) είτε προς την 5n^4+10n^3-6n^2-11n+1>0 (αληθής για n>1). Το μόνο λοιπόν που απομένει είναι να παρατηρήσουμε ότι δεν έχει ακέραια λύση για n=1 η m^5-n^5=16mn (άμεσο).


Γιώργος Μπαλόγλου -- κρυσταλλογράφω άρα υπάρχω

Ὁρᾷς, τὸ κάλλος ὅσσον ἐστὶ τῆς λίθου, ἐν ταῖς ἀτάκτοις τῶν φλεβῶν εὐταξίαις. -- Παλατινή Ανθολογία 9.695 -- Ιδού του πετραδιού η άμετρη ομορφιά, μεσ' των φλεβών τις άναρχες πειθαρχίες.
Απάντηση

Επιστροφή σε “Θέματα διαγωνισμών (ΕΜΕ, ΚΥΜΕ, BMO, JBMO, IMO, Kangaroo κλπ)”

Μέλη σε σύνδεση

Μέλη σε αυτήν τη Δ. Συζήτηση: Δεν υπάρχουν εγγεγραμμένα μέλη και 11 επισκέπτες